Packrat 5 Study guides, Class notes & Summaries

Looking for the best study guides, study notes and summaries about Packrat 5? On this page you'll find 31 study documents about Packrat 5.

Page 3 out of 31 results

Sort by

Neurology PACKRAT Exam - Questions, Answers and Explanations (Complete Solutions)
  • Neurology PACKRAT Exam - Questions, Answers and Explanations (Complete Solutions)

  • Exam (elaborations) • 32 pages • 2024
  • Neurology PACKRAT Exam - Questions, Answers and Explanations (Complete Solutions) History & Physical/Neurology A 73 year-old male presents to the clinic with his wife. His wife has noticed that he has developed a resting tremor in his right hand and a shuffling gait over the last year. What finding on physical examination would support your suspected diagnosis? A. Chorea B. Dystonia C. Masked facies D. Hyperreflexia Explanation The patient symptoms are consistent with Parkinsonism. Physical ex...
    (0)
  • $25.99
  • + learn more
Cardiology PACKRAT Exam - Questions, Answers and Explanations (Complete Solutions)
  • Cardiology PACKRAT Exam - Questions, Answers and Explanations (Complete Solutions)

  • Exam (elaborations) • 86 pages • 2024
  • Cardiology PACKRAT Exam - Questions, Answers and Explanations (Complete Solutions) History & Physical/Cardiology Which of the following conditions would cause a positive Kussmaul's sign on physical examination? A. Left ventricular failure B. Pulmonary edema C. Coarctation of the aorta D. Constrictive pericarditis Explanation A. Left ventricular failure results in the back-up of blood into the left atrium and then the pulmonary system so it would not be associated with Kussmaul's sign. B. Pul...
    (0)
  • $30.49
  • + learn more
PACKRAT Exam Questions with Correct Answers.docx
  • PACKRAT Exam Questions with Correct Answers.docx

  • Exam (elaborations) • 45 pages • 2024
  • PACKRAT Exam Questions with Correct A 23 yo male with recent URI sxs presents c/o chest pain. His pain is worse when lying down and better sitting up and leaning forward. ECG shows widespread ST segment elevation. Dx - Answer-Pericardial friction rub 75 yo woman presents to the office with complain of vision loss. Exam reveals a palpable cord in the temporal region. Most helpful initial test to order - Answer-ESR Recommended LDL tor educe risk of CAD in a diabetic pt - Answer-100mg/dL ...
    (0)
  • $13.49
  • + learn more
PACKRAT OBGYN Exam Questions with Correct Answers.docx
  • PACKRAT OBGYN Exam Questions with Correct Answers.docx

  • Exam (elaborations) • 45 pages • 2024
  • PACKRAT OBGYN Exam Questions with Correct A 1.History & Physical/Obstetrics/Gynecology A 26 year-old monogamous female presents with cyclic pelvic pain that has been increasing over the last 6 months. She complains of significant dysmenorrhea and dyspareunia. She uses condoms for birth control. On physical examination her uterus is retroverted and non-mobile, and she has a palpable adnexal mass on the left side. Her serum pregnancy test is negative. Which of the following is the most likely ...
    (0)
  • $12.49
  • + learn more
PACKRAT – OBGYN QUESTIONS AND ANSWERS 2023
  • PACKRAT – OBGYN QUESTIONS AND ANSWERS 2023

  • Exam (elaborations) • 45 pages • 2023
  • PACKRAT – OBGYN QUESTIONS AND ANSWERS 2023 1. History & Physical/Obstetrics/Gynecology A 26 year-old monogamous female presents with cyclic pelvic pain that has been increasing over the last 6 months. She complains of significant dysmenorrhea and dyspareunia. She uses condoms for birth control. On physical examination her uterus is retroverted and non-mobile, and she has a palpable adnexal mass on the left side. Her serum pregnancy test is negative. Which of the following is the most likely ...
    (0)
  • $21.49
  • + learn more
PACKRAT 14 Exam Questions with Correct Answers.docx
  • PACKRAT 14 Exam Questions with Correct Answers.docx

  • Exam (elaborations) • 77 pages • 2024
  • PACKRAT 14 Exam Questions with Correct A When performing a pre-participation sports physical in the adolescent population, a murmur with which of the following qualities indicates a risk for sudden death during exercise? A. Increases with the Valsalva maneuver B. Increases with squatting maneuver C. Associated with a mid-systolic click D. Mid-systolic without radiation to the carotids - Answer-(c) A. Hypertrophic cardiomyopathy (HCM) is a known cause of sudden death during or just after phy...
    (0)
  • $13.49
  • + learn more
PACKRAT Exam Study Guide  100% Correct Answers  Verified  Latest 2024 Version
  • PACKRAT Exam Study Guide 100% Correct Answers Verified Latest 2024 Version

  • Exam (elaborations) • 139 pages • 2024
  • PACKRAT Exam Study Guide | 100% Correct Answers | Verified | Latest 2024 Version 1. Health Maintenance/Obstetrics/Gynecology Which of the following portends the greatest risk for the development of breast cancer? A. Nulliparity B. Early menarche C. BRCA 1 positivity D. Maternal family history of breast cancer - (u) A. See C for explanation. (u) B. See C for explanation. (c) C. BRCA1 positivity is associated with half of the early onset breast cancers and 90% of the hereditary ovarian ...
    (0)
  • $9.49
  • + learn more
Cardiology PACKRAT Questions  100% Correct Answers  Verified Latest 2024 Version
  • Cardiology PACKRAT Questions 100% Correct Answers Verified Latest 2024 Version

  • Exam (elaborations) • 145 pages • 2024
  • Cardiology PACKRAT Questions | 100% Correct Answers | Verified | Latest 2024 Version 1. History & Physical/Cardiology Which of the following conditions would cause a positive Kussmaul's sign on physical examination? Answers A. Left ventricular failure B. Pulmonary edema C. Coarctation of the aorta D. Constrictive pericarditis - (u) A. Left ventricular failure results in the back-up of blood into the left atrium and then the pulmonary system so it would not be associated with Kussmaul...
    (0)
  • $9.49
  • + learn more
PACKRAT 12 Questions bank, All Questions and answers, 2022. Rated A+
  • PACKRAT 12 Questions bank, All Questions and answers, 2022. Rated A+

  • Exam (elaborations) • 99 pages • 2023
  • PACKRAT 12 Questions bank, All Questions and answers, 2022. Rated A+ Document Content and Description Below PACKRAT 12 Questions bank, All Questions and answers, 2022. Rated A+ Which of the following is a major contraindication to curative surgical resection of a lung tumor? A. Liver metastases B. Vagus nerve involvement C. Non-malignant pleural effusion D. Chest wall invasion of the tumor - (c) A. Distant metastases, except for solitary brain and adrenal metastases are an absolute contraindic...
    (0)
  • $12.49
  • + learn more